Você está na página 1de 13

Teorema de

Stolz–Cesàro

Danilo e Lenilson
12 Jan. 2023
Sumário

Parte 1 Teoria Page 2


1.1 Teorema e Corolários 2
1.2 Problemas 3

Parte 2 Soluções Page 5


2.1 Referências bibliográficas 11

1
Parte 1

Teoria

Vamos falar a seguir sobre o importante Teorema de Stolz–Cesàro e algumas consequências imediatas dele, não va-
mos entrar em detalhes da demonstração, mas para curiosidade visite a referência bibliográfica na última página.
Também vamos resolver 13 problemas em que tal teorema é aplicável. As siglas usadas quer dizer:
MR - Mathematical Reflections
PUTNAM - William Lowell Putnam Mathematics Competition
OBMU - Olimpı́ada Brasileira de Matemática Nı́vel Universitário
RMC - Romanian Mathematical Competitions

1.1 Teorema e Corolários


Teorema 1.1.1: Stolz–Cesàro

Sejam (𝑎 𝑛 )𝑛 ⩾1 e (𝑏 𝑛 )𝑛 ⩾1 duas sequências de números reais. Supondo que 𝑏 𝑛 é uma sequência estritamente
𝑎 𝑛+1 − 𝑎 𝑛
monótona, ilimitada e que o limite lim existe, então
𝑛→∞ 𝑏 𝑛+1 − 𝑏 𝑛

𝑎𝑛 𝑎 𝑛+1 − 𝑎 𝑛
lim = lim .
𝑛→∞ 𝑏𝑛 𝑛→∞ 𝑏 𝑛+1 − 𝑏 𝑛

Note:-
Tal teorema vale quando 𝑏 𝑛 é estritamente crescente e diverge +∞, ou estritamente decrescente e diverge para
−∞. Mas não apenas nesses dois casos, como também vale se (𝑎 𝑛 ) → 0 e (𝑏 𝑛 ) → 0, com 𝑏 𝑛 estritamente
decrescente, é o famoso caso 0/0.

Corolário 1.1.1
Dada qualquer sequência (𝑥 𝑛 )𝑛 ⩾1 de números reais, suponha que lim 𝑥 𝑛 existe (finito ou infinito), então
𝑛→∞

𝑥1 + · · · + 𝑥 𝑛
lim = lim 𝑥 𝑛 .
𝑛→∞ 𝑛 𝑛→∞

Corolário 1.1.2
Dada qualquer sequência (𝑥 𝑛 )𝑛 ⩾1 de números reais positivos, suponha que lim 𝑥 𝑛 existe (finito ou infinito),
𝑛→∞
então √
lim 𝑛
𝑥1 · · · 𝑥 𝑛 = lim 𝑥 𝑛 .
𝑛→∞ 𝑛→∞

2
Corolário 1.1.3
𝑦𝑛+1
Dada qualquer sequência (𝑦𝑛 )𝑛 ⩾1 de números reais positivos, suponha que lim existe (finito ou
𝑛→∞ 𝑦𝑛
infinito), então
√ 𝑦𝑛+1
lim 𝑛
𝑦𝑛 = lim .
𝑛→∞ 𝑛→∞ 𝑦𝑛

Veja alguns exemplos de aplicação do Corolário 1.1.3:

Exemplo 1.1.1
√ 𝑛+1
lim 𝑛 𝑛 = lim = 1.
𝑛→∞ 𝑛→∞ 𝑛

Exemplo 1.1.2

(𝑛 + 1)! · 𝑛 𝑛 𝑛𝑛
𝑛
r
𝑛! 𝑛 𝑛! 1 1
lim = lim = lim = lim = lim  = .
𝑛→∞ 𝑛 𝑛→∞ 𝑛 𝑛 𝑛→∞ 𝑛! · (𝑛 + 1)𝑛+1 𝑛→∞ (𝑛 + 1)𝑛 𝑛→∞ 1 + 1 𝑛 𝑒
𝑛

1.2 Problemas
1. (Competição Elon Lages Lima 2020) O limite
Í 
𝑛 1
𝑘=1 𝑘 1/𝑘
lim
𝑛→∞ 𝑛
é dado por:
(a) 1
(b) 2
(c) 𝜋
(d) 𝜋2 /6
(e) 𝑒

2. (MR 2019) Calcule


𝜋2 Í𝑛 1
6 − 𝑘=1 𝑘 2
lim .
𝑛→∞ + 𝑛1

ln 1

3. (MR 2020) Calcule


1 𝑛
2
sen 1 + sen 21

+ · · · + sen 𝑛
lim 1 1 1
.
𝑛→∞ + +···+
1! 2! 𝑛!

Õ 1 1 1 𝜋2 Õ 1 1 1
4. (OBMU 2003) Sabemos que =1+ + + · · · = . Defina 𝑓 (𝑛) = = 1 + 2 + ··· + 2.
𝑘2 22 32 6 𝑘2 2 𝑛
𝑘>0 0<𝑘 ⩽ 𝑛
Prove que existe um número real 𝑎 > 0 tal que existe o limite:

𝜋2 𝑎
 
lim 𝑓 (𝑛) − + · 𝑛2 .
𝑛→∞ 6 𝑛

Calcule 𝑎 e este limite.

3
5. (MR 2020) Calcule
𝑛
p
12 32 · · · (2𝑛 + 1)2
lim .
𝑛→∞ 1 + 3 + · · · + (2𝑛 + 1)

6. (PUTNAM 1966) Defina a sequência 𝑎 𝑛 por 𝑎 1 ∈ (0, 1), e 𝑎 𝑛+1 = 𝑎 𝑛 (1 − 𝑎 𝑛 ). Mostre que lim 𝑛𝑎 𝑛 = 1.
𝑛→∞

7. (OBMU 2003) Defina 𝑎1 = 3, 𝑎 𝑛+1 = 𝑎 𝑛2 − 2.


log log 𝑎 𝑛
Prove que lim = log 2 e calcule lim (log log 𝑎 𝑛 − 𝑛 log 2).
𝑛→∞ 𝑛 𝑛→∞
(Observação: os logaritmos estão todos na base 𝑒).

8. (PUTNAM 2012) Suponha que 𝑎0 = 1 e que 𝑎 𝑛+1 = 𝑎 𝑛 + 𝑒 −𝑎 𝑛 para 𝑛 = 0, 1, 2, · · · . Então (𝑎 𝑛 − ln 𝑛) tem


um limite finito quando 𝑛 → ∞?

1
9. (RMC 2004) Seja 𝑥0 > 0 e para qualquer inteiro positivo 𝑛, considere 𝑥 𝑛+1 = 𝑥 𝑛 + √ . Encontre:
𝑥𝑛
(a) lim 𝑥 𝑛 ;
𝑛→∞
𝑥 𝑛3
(b) lim .
𝑛→∞ 𝑛2

10. Seja 𝑓 : ℝ → ℝ uma função contı́nua tal que a sequência (𝑎 𝑛 )𝑛 ⩾0 definida por
∫ 1
𝑎𝑛 = 𝑓 (𝑛 + 𝑥) 𝑑𝑥
0

é convergente. Prove que a sequência (𝑏 𝑛 )𝑛 ⩾0 , dada por


∫ 1
𝑏𝑛 = 𝑓 (𝑛𝑥) 𝑑𝑥
0

também é convergente.

Í 𝑎𝑛
11. (OBMU 2016 / VJIMC 2006) Seja (𝑎 𝑛 )𝑛 ⩾1 uma sucessão de números reais tal que 𝑛 ⩾1 𝑛 converge. Prove
que
𝑛

lim 𝑎 𝑘 = 0.
𝑛→∞ 𝑛
𝑘=1

12. (PUTNAM 2006) Seja 𝑘 um número inteiro maior que 1. Suponha que 𝑎0 > 0, e defina
1
𝑎 𝑛+1 = 𝑎 𝑛 + √𝑘
𝑎𝑛
para 𝑛 > 0. Calcule
𝑎 𝑛𝑘+1
lim .
𝑛→∞ 𝑛𝑘
∫ 𝑥
13. (PUTNAM 2008) Seja 𝐹0 (𝑥) = ln (𝑥). Para 𝑛 ⩾ 0 e 𝑥 > 0, seja 𝐹𝑛+1 (𝑥) = 𝐹𝑛 (𝑡) 𝑑𝑡. Calcule o limite
0

𝑛!𝐹𝑛 (1)
lim .
𝑛→∞ ln (𝑛)

4
Parte 2

Soluções

1. Aplicando o Corolário 1.1.1:


Í 
𝑛 1 1 1 1
𝑘=1 𝑘 1/𝑘 1 + 21/2
+···+ 𝑛 1/𝑛 1
lim = lim = lim .
𝑛→∞ 𝑛 𝑛→∞ 𝑛 𝑛→∞ 𝑛 1/𝑛
1
Resta então descobrirmos o valor de lim . Observe que:
𝑛→∞ 𝑛 1/𝑛
1 ln 𝑛
= lim 𝑒 ln 𝑛
−1/𝑛 1
lim = 𝑒 lim𝑛→∞ − 𝑛 = 𝑒 lim𝑛→∞ − 𝑛 = 𝑒 0 = 1.
𝑛→∞ 𝑛 1/𝑛 𝑛→∞
Na igualdade em vermelho aplicamos a regra de L’Hôpital’s. Alternativa A é a resposta.

2. Observe    
𝜋2 Í𝑛 𝜋2 Í𝑛
𝜋2

Í𝑛 1 𝑛 6 − 1
𝑘=1 𝑘 2 𝑛 6 − 1
𝑘=1 𝑘 2
6 𝑘=1 𝑘 2
lim = lim = lim 1 𝑛

𝑛→∞ + 𝑛1 𝑛→∞ 𝑛 · ln 1 + 1 𝑛→∞
  
ln 1 𝑛 ln 1 + 𝑛
 Í𝑛 
𝜋2
𝑛 6 − 1
𝑘=1 𝑘 2
𝜋2

Í𝑛 1
6 𝑘=1 𝑘 2
⇔ lim = lim 1
𝑛→∞ 1 𝑛→∞
𝑛
Esse é o caso em que as duas sequências (no numerador e no denominador) ambas tendem a zero, então por
Stolz–Cesàro segue que:
Í𝑛+1  Í𝑛  h i
𝜋2 1 𝜋2 1 1
6 − 𝑘=1 𝑘 2 − 6 − 𝑘=1 𝑘 2 − (𝑛+11)2 (𝑛+1)2 𝑛
lim = lim = lim h i = lim = 1.
𝑛→∞ 1
𝑛+1 − 1
𝑛
𝑛→∞ 1
𝑛+1 − 1
𝑛
𝑛→∞ 1 𝑛→∞ 𝑛+1
𝑛(𝑛+1)

1
3. Multiplicando por 𝑛 em cima e em baixo e usando o teorema de Stolz–Cesàro (especificamente o Corolário
1.1.1) segue que:
Í𝑛 𝑘 1 𝑘 Í𝑛 𝑛
sen 1𝑘 1
sen 𝑛1
 
𝑘=1 𝑘=1 sen 𝑘 𝑛
lim Í𝑛 1 = lim 1 Í𝑛 1
= lim 1

𝑛→∞ 𝑛→∞ 𝑛→∞
𝑘=1 𝑘 ! 𝑛 𝑘=1 𝑘 ! 𝑛!
𝑛 𝑛
sen 𝑛1 · 𝑛 𝑛 sen 𝑛1 𝑛!  sen 𝑛  1𝑛 𝑛!

⇔ lim = lim · = lim · lim .
𝑛→∞
𝑛! · 𝑛
1 𝑛 𝑛→∞ 1
𝑛
𝑛 𝑛 𝑛→0+ 𝑛 𝑛→∞ 𝑛𝑛
 sen 𝑛  1𝑛
Quanto será que vale lim+ ? Vejamos:
𝑛→0 𝑛
 sen 𝑛  1𝑛 ln ( sen 𝑛
𝑛 )
lim+ = 𝑒 lim𝑛→0+ 𝑛 .
𝑛→0 𝑛
5
Calculando o limite que está no expoente (em diversas etapas utilizamos L’Hôpital):
sen 𝑛

ln 𝑛 𝑛 (cos 𝑛) · 𝑛 − sen 𝑛 (−sen 𝑛) · 𝑛 + cos 𝑛 − cos 𝑛
lim = lim+ · = lim+ = 0.
𝑛→0+ 𝑛 𝑛→0 sen 𝑛 𝑛 2 𝑛→ 0 2𝑛
Logo
 sen 𝑛  1𝑛
lim+ = 𝑒 0 = 1.
𝑛→0 𝑛
𝑛!
Resta então descobrirmos quanto vale lim . Observe que:
𝑛→∞ 𝑛𝑛
𝑛! 1 2 𝑛−1 𝑛
𝑛
= · ··· · .
𝑛 𝑛 𝑛 𝑛 𝑛
𝑛!
E claramente 𝑛 𝑛 > 𝑛! para 𝑛 > 1, logo lim = 0, e portanto o limite do nosso problema é 0.
𝑛→∞ 𝑛𝑛

4. Mexendo um pouco no nosso limite obtemos:


𝜋2 𝑎
!
𝑓 (𝑛) − 6 + 𝑛
lim 1
.
𝑛→∞
𝑛2

Chegamos no caso em que tanto o numerador e o denominador tendem a zero, podendo aplicar Stolz-Cesàro:
Í𝑛+1 Í 
1 𝜋2 𝑎 𝑛 1 𝜋2 𝑎 𝑎 𝑎
− + − − + 1
+
𝑘=1 𝑘 2 6 𝑛+1 𝑘=1 𝑘 2 6 𝑛 (𝑛+1)2 𝑛+1 − 𝑛 𝑎(𝑛 + 1)2 − 𝑎(𝑛 + 1)𝑛 − 𝑛
lim 1 1
= lim 2𝑛+1
= lim 2𝑛+1

𝑛→∞ − 𝑛→∞ − (𝑛+ 𝑛→∞
(𝑛+1)2 𝑛2 1)2 𝑛 2 𝑛

𝑎𝑛 + 𝑎 − 𝑛 𝑛 2 (𝑎 − 1) + 𝑎𝑛
⇔ lim 2𝑛+1
= lim .
𝑛→∞ 𝑛→∞ 2𝑛 + 1
𝑛
O problema requer que o limite exista como número real finito, caso 𝑎 > 1 o limite tende a +∞ e quando 𝑎 < 1
o limite tende a −∞, isso porque o polinômio do numerador possuiria grau maior que o do denominador,
então a única possibilidade é 𝑎 = 1. Vejamos quanto ficaria o limite:
𝑛 𝑛 1 1
lim = lim  = lim = .
𝑛→∞ 2𝑛 + 1 𝑛→∞ 𝑛 2 + 𝑛1 𝑛→∞ 2 + 1
𝑛
2

Perfeito.

5. Sabendo que 1 + 3 + · · · + (2𝑛 + 1) = (𝑛 + 1)2 , então:


s
𝑛
p
12 32 · · · (2𝑛 + 1)2 𝑛 ((2𝑛 + 1)!)2
lim = lim .
𝑛→∞ 1 + 3 + · · · + (2𝑛 + 1) 𝑛→∞ (𝑛 + 1)2𝑛 · (2𝑛 · 𝑛!)2

Como estamos tratando de números reais positivos, podemos utilizar o Corolário 1.1.3:
 
s ((2𝑛+3)!)2
(2𝑛 + 3)2 · (2𝑛 + 2)2 · (𝑛 + 1)2𝑛
2
𝑛 ((2𝑛 + 1)!)2 (𝑛+2)2(𝑛+1) ·(2(𝑛+1) ·(𝑛+1)!)
lim = lim = lim ⇔
𝑛→∞ (𝑛 + 1)2𝑛 · (2𝑛 · 𝑛!)2 𝑛→∞ 𝑛→∞ 4 · (𝑛 + 1)2 · (𝑛 + 2)2 · (𝑛 + 2)2𝑛
 
((2𝑛+1)!)2
(𝑛+1)2𝑛 ·(2𝑛 ·𝑛 !)2

2   2𝑛 2  2𝑛
𝑛+1 𝑛+1
  
2𝑛 + 3 2𝑛 + 3
⇔ lim = lim · lim .
𝑛→∞ 𝑛+2 𝑛+2 𝑛→∞ 𝑛+2 𝑛→∞ 𝑛+2
| {z }
=4

6
 2𝑛
𝑛+1

Resta então descobrirmos quanto vale lim . Vejamos:
𝑛→∞ 𝑛+2
2
© ª
 2𝑛 𝑛2  −2−𝑡 ! 2  −𝑡  −2 ®
𝑛+1
    ­  
1 1 ­ 1 1 ® = 1.
®
lim = lim 1 − = lim 1+ = ­­ lim 1 + · lim 1 −
𝑛→∞ 𝑛+2 𝑛→∞ 𝑛+2 𝑡→−∞ 𝑡 ­𝑡→−∞ 𝑡 𝑡→∞ 𝑡 ®
® 𝑒2
­| {z } | {z }®
= 1𝑒 =1
« ¬
Terminamos afirmando:
𝑛
p
12 32 · · · (2𝑛 + 1)2 4
lim = 2.
𝑛→∞ 1 + 3 + · · · + (2𝑛 + 1) 𝑒

6. Observa-se que 𝑎 𝑛+1 − 𝑎 𝑛 = −𝑎 𝑛2 < 0 ⇒ 𝑎 𝑛+1 < 𝑎 𝑛 . Logo 𝑎 𝑛 é monótona decrescente. Em particular,
𝑎 𝑛 < 𝑎 1 < 1. Veja que 𝑎1 > 0, e supondo que 𝑎 𝑛 > 0, com efeito é fácil verificar que 𝑎 𝑛+1 = 𝑎 𝑛 (1 − 𝑎 𝑛 ) > 0,
|{z} | {z }
>0 >0
logo (𝑎 𝑛 )𝑛 ⩾1 é limitada pois 0 < 𝑎 𝑛 < 1, e portanto convergente. Seja lim 𝑎 𝑛 = ℓ , então
𝑛→∞

lim 𝑎 𝑛+1 = lim 𝑎 𝑛 − 𝑎 𝑛2 ⇒ ℓ = ℓ − ℓ 2 ⇒ ℓ = 0.


𝑛→∞ 𝑛→∞

1 𝑛
Concluı́mos que 𝑎 𝑛 converge para zero. Seja então 𝑏 𝑛 = ⇒ lim 𝑏 𝑛 = ∞. Chamando 𝑥 𝑛 = 𝑛 · 𝑎 𝑛 = ,
𝑎𝑛 𝑛→∞ 𝑏𝑛
podemos usar o Teorema de Stolz-Cesàro:
𝑛+1−𝑛 1 1
lim 𝑥 𝑛 = lim = lim = lim = lim 1 − 𝑎 𝑛 = 1
𝑛→∞ 𝑛→∞ 𝑏 𝑛+1 − 𝑏 𝑛 𝑛→∞ 1
𝑎 𝑛+1 −
1
𝑎𝑛
𝑛→∞ 1
𝑎 𝑛 (1−𝑎 𝑛 )
− 1
𝑎𝑛
𝑛→∞

∴ lim 𝑛 · 𝑎 𝑛 = 1.
𝑛→∞

7. Primeiramente vamos provar que 𝑎 𝑛 > 2 para 𝑛 ⩾ 1. Como 𝑎1 = 3 ⇒ 𝑎 2 = 32 −2 = 7, que são os casos iniciais,
e suponhamos por hipótese de indução que 𝑎 𝑛 > 2. Dessa forma, pela hipótese, 𝑎 𝑛+1 = 𝑎 𝑛2 − 2 > 22 − 2 = 2 ⇒
𝑎 𝑛+1 > 2, provando que também é válido para 𝑎 𝑛+1 . Logo, 𝑎 𝑛+1 − 𝑎 𝑛 = 𝑎 𝑛2 − 𝑎 𝑛 − 2 ⇔ 𝑎 𝑛+1 − 𝑎 𝑛 > 4 − 2 − 2 = 0,
então 𝑎 𝑛+1 − 𝑎 𝑛 > 0, significando que 𝑎 𝑛 é estritamente crescente. Supondo que 𝑎 𝑛 converge para um valor
real finito, então seja ℓ = lim 𝑎 𝑛 , logo:
𝑛→∞

lim 𝑎 𝑛+1 = lim 𝑎 𝑛2 − 2 ⇔ ℓ = ℓ 2 − 2 ⇔ ℓ = −1 ou ℓ = 2.


𝑛→∞ 𝑛→∞

Absurdo pois 𝑎1 já é maior que esses dois valores. Então, 𝑎 𝑛 diverge para +∞. Pensando agora no nosso
limite (aqui vamos usar ln como notação para o logaritmo natural, diferente do que faz o enunciado), como
𝑛 → ∞ e é estritamente crescente, então podemos utilizar o Teorema de Stolz-Cesàro:

ln 𝑎 𝑛2 − 2

ln ln 𝑎 𝑛
  
ln (ln (𝑎 𝑛+1 )) − ln (ln (𝑎 𝑛 )) ln (𝑎 𝑛+1 )
lim = lim = lim ln = lim ln = ln 2.
𝑛→∞ 𝑛 𝑛→∞ 𝑛+1−𝑛 𝑛→∞ ln (𝑎 𝑛 ) 𝑛→∞ ln (𝑎 𝑛 )

ln 𝑎 𝑛2 − 2

Precisamos provar que lim = 2, e como lim ln (𝑎 𝑛 + 𝑘) = lim ln (𝑎 𝑛 ) = +∞ para qualquer 𝑘
𝑛→∞ ln (𝑎 𝑛 ) 𝑛→∞ 𝑛→∞
real, então: h √  √ i  √   √ 
ln 𝑎 𝑛 − 2 𝑎 𝑛 + 2 ln 𝑎 𝑛 − 2 ln 𝑎 𝑛 + 2
lim = lim + ⇔
𝑛→∞ ln (𝑎 𝑛 ) 𝑛→∞ ln (𝑎 𝑛 ) ln (𝑎 𝑛 )
 √   √ 
ln 𝑎 𝑛 − 2 ln 𝑎 𝑛 + 2
⇔ lim + lim = 1 + 1 = 2.
𝑛→∞ ln (𝑎 𝑛 ) 𝑛→∞ ln (𝑎 𝑛 )
7
ln (ln 𝑎 𝑛 )
Provado a primeira parte. Então calculando o limite lim (ln (ln 𝑎 𝑛 ) − 𝑛 ln 2), como ln 2 = lim ,
𝑛→∞ 𝑛→∞ 𝑛
temos que:
ln (ln 𝑎 𝑛 )
  
lim (ln (ln 𝑎 𝑛 ) − 𝑛 ln 2) = lim ln (ln 𝑎 𝑛 ) − 𝑛 · = lim (ln (ln 𝑎 𝑛 ) − ln (ln 𝑎 𝑛 )) = 0.
𝑛→∞ 𝑛→∞ 𝑛 𝑛→∞

Terminamos o problema.

8. Verificando a monotonicidade de 𝑎 𝑛 :
𝑎 𝑛+1 = 𝑎 𝑛 + 𝑒 −𝑎 𝑛 ⇒ 𝑎 𝑛+1 − 𝑎 𝑛 = 𝑒 −𝑎 𝑛 > 0 ∴ 𝑎 𝑛+1 > 𝑎 𝑛 .
Portanto a sequência 𝑎 𝑛 é estritamente crescente. Agora, suponhamos que 𝑎 𝑛 é convergente e que lim 𝑎 𝑛 =
𝑛→∞
ℓ < ∞, então:
lim 𝑎 𝑛+1 = lim 𝑎 𝑛 + 𝑒 −𝑎 𝑛 ⇒ ℓ = ℓ + 𝑒 −ℓ ⇒ 𝑒 −ℓ = 0.
𝑛→∞ 𝑛→∞
Um absurdo, então 𝑎 𝑛 diverge para +∞.
𝑒 𝑎𝑛
Tomando 𝑏 𝑛 = 𝑎 𝑛 − ln 𝑛 e 𝑐 𝑛 = 𝑒 𝑏 𝑛 = 𝑒 𝑎 𝑛 −ln 𝑛 = 𝑒 𝑎 𝑛 · 𝑒 ln 𝑛 =
1
. Vamos aplicar o teorema de Stolz-Cesàro
𝑛
em 𝑐 𝑛 :
𝑒 𝑎𝑛 𝑒 𝑎 𝑛+1 − 𝑒 𝑎 𝑛 −𝑎 𝑛
 −𝑎
𝑛

lim = lim = lim 𝑒 𝑎 𝑛+1 − 𝑒 𝑎 𝑛 = lim 𝑒 𝑎 𝑛 +𝑒 − 𝑒 𝑎 𝑛 = lim 𝑒 𝑎 𝑛 𝑒 𝑒 − 1 .
𝑛→∞ 𝑛 𝑛→∞ 𝑛 + 1 − 𝑛 𝑛→∞ 𝑛→∞ 𝑛→∞
−𝑎 𝑛
Como já vimos que 𝑒 −𝑎 𝑛 → 0 ⇒ 𝑒 𝑒 → 1, afinal 𝑎 𝑛 diverge para +∞. Logo, o nosso limite vale:
−𝑎 𝑛 −𝑎 𝑛
 −𝑎 𝑛
 𝑒𝑒 −1 𝑒𝑒 −1
lim 𝑒 𝑎 𝑛 𝑒 𝑒 − 1 = lim = −𝑎lim = 1.
𝑛→∞ 𝑛→∞ 𝑒 −𝑎 𝑛 𝑒 𝑛 →0 𝑒 −𝑎 𝑛
∴ 𝑐 𝑛 → 1.
Aplicando o logaritmo:
 
lim 𝑐 𝑛 = lim 𝑒 𝑏 𝑛 = lim 𝑒 𝑎 𝑛 −ln 𝑛 = 1 ⇔ ln lim 𝑒 𝑎 𝑛 −ln 𝑛 = ln 1 ⇔ lim (𝑎 𝑛 − ln 𝑛) = ln 1 = 0.
𝑛→∞ 𝑛→∞ 𝑛→∞ 𝑛→∞ 𝑛→∞

Provando que (𝑎 𝑛 − ln 𝑛) tem um limite finito quando 𝑛 → ∞.

9. Como 𝑥 𝑛 > 0 para qualquer 𝑛 inteiro não negativo, então 𝑥 𝑛+1 − 𝑥 𝑛 = √1𝑥 > 0 ⇒ 𝑥 𝑛+1 > 𝑥 𝑛 , portanto
𝑛
(𝑥 𝑛 )𝑛 ⩾0 é uma sequência estritamente crescente. Suponhamos que 𝑥 𝑛 converge para um valor 𝐿 ∈ ℝ, então:
1 1 1
lim 𝑥 𝑛+1 = lim 𝑥 𝑛 + √ ⇒ 𝐿 = 𝐿 + √ ⇒ √ = 0.
𝑛→∞ 𝑛→∞ 𝑥𝑛 𝐿 𝐿
Temos um absurdo, e dessa forma 𝑥 𝑛 diverge para ∞. Concluindo que lim 𝑥 𝑛 = ∞. Resolvida a letra (a).
𝑛→∞
Como 𝑛 → ∞ e a sequência 𝑏 𝑛 = 𝑛 é estritamente crescente, podemos utilizar o teorema de Stolz:
q p
𝑥 𝑛+
3
1− 𝑥 𝑛3 𝑥 3 − 𝑥 𝑛3
p
𝑥 𝑛3
ℓ = lim = lim = lim q 𝑛+1 p
𝑛→∞ 𝑛 𝑛→∞ 𝑛+1−𝑛 𝑛→∞
𝑥 𝑛+
3
1 + 𝑥𝑛
3

(𝑥 𝑛+1 − 𝑥 𝑛 ) 𝑥 𝑛+
2
1 + 𝑥 𝑛+1 𝑥 𝑛 + 𝑥 𝑛
2
𝑥 𝑛+1 + 𝑥 𝑛+1 𝑥 𝑛 + 𝑥 𝑛
 2 2
⇒ ℓ = lim 3 3
= lim √  2 3 3
.
𝑛→∞ 𝑛→∞
 
𝑥 𝑛+1 + 𝑥 𝑛
2 2
𝑥 𝑛 𝑥 𝑛+1 + 𝑥 𝑛2
𝑥 𝑛+1
Dividindo o numerador e o denominador por 𝑥 𝑛2 , e além disso sabendo que lim = 1, obtemos
𝑛→∞ 𝑥𝑛
2
𝑥 𝑛+1 𝑥 𝑛+1

+ +1
𝑥𝑛 𝑥𝑛 3
ℓ = lim  32 = .
𝑛→∞ 2
𝑥 𝑛+1

+1
𝑥𝑛
8
Dessa forma,
p !2  2
𝑥3 𝑥 𝑛3 3 9
lim 𝑛2 = lim = = ,
𝑛→∞ 𝑛 𝑛→∞ 𝑛 2 4

é a resposta da letra (b).

10. Consideremos a primitiva ∫ 𝑡


𝐹(𝑡) = 𝑓 (𝑥) 𝑑𝑥.
0
Então ∫ 𝑛
𝑡 𝑓 (𝑥) 𝑑𝑥
𝐹(𝑛𝑡) 1
𝐹(𝑛)
∫ ∫
𝑓 (𝑛𝑥) 𝑑𝑥 = ⇒ 𝑓 (𝑛𝑥) 𝑑𝑥 = = 0
.
0 𝑛 0 𝑛 𝑛
Então a sequência 𝑏 𝑛 do nosso problema pode ser reescrita como
∫ 𝑛
𝑓 (𝑥) 𝑑𝑥
𝑏𝑛 = 0
.
𝑛
Queremos calcular lim 𝑏 𝑛 , que por Stolz-Cesàro:
𝑛→∞
∫ 𝑛 ∫ 𝑛+1 ∫ 𝑛
𝑓 (𝑥) 𝑑𝑥 𝑓 (𝑥) 𝑑𝑥 − 𝑓 (𝑥) 𝑑𝑥
lim 𝑏 𝑛 = lim 0
= lim 0 0
𝑛→∞ 𝑛→∞ 𝑛 𝑛→∞ (𝑛 + 1) − 𝑛
∫ 𝑛+1 ∫ 1
= lim 𝑓 (𝑥) 𝑑𝑥 = lim 𝑓 (𝑛 + 𝑥) 𝑑𝑥 = lim 𝑎 𝑛 .
𝑛→∞ 𝑛 𝑛→∞ 0 𝑛→∞

Como 𝑎 𝑛 converge, segue que 𝑏 𝑛 converge para o mesmo valor.

11. Da soma telescópica obtemos:


𝑛
Õ
𝑎 𝑘+1 𝑏 𝑘+1 − 𝑎 𝑘 𝑏 𝑘 = 𝑎 𝑛+1 𝑏 𝑛+1 − 𝑎 𝑚 𝑏 𝑚
𝑘=𝑚

𝑛
Õ 𝑛
Õ
⇔ 𝑎 𝑘 (𝑏 𝑘+1 − 𝑏 𝑘 ) = [𝑎 𝑛+1 𝑏 𝑛+1 − 𝑎 𝑚 𝑏 𝑚 ] − (𝑎 𝑘+1 − 𝑎 𝑘 ) 𝑏 𝑘+1 .
𝑘=𝑚 𝑘=𝑚
𝑛
Õ 𝑎𝑘
Seja a sequência 𝑏 𝑛 = , e veja que lim 𝑏 𝑛 = lim 𝑏 𝑛+1 = 𝑏 com 𝑏 real, pois a série converge. Então
𝑘 𝑛→∞ 𝑛→∞
𝑘=1
temos que 𝑎 𝑛+1 = (𝑛 + 1) (𝑏 𝑛+1 − 𝑏 𝑛 ), e dessa forma
𝑛 𝑛 𝑛
! !
1Õ 1 Õ 1 Õ
ℓ = lim 𝑎 𝑘 = lim 𝑎1 + 𝑎 𝑘+1 = lim 𝑎1 + (𝑘 + 1) (𝑏 𝑘+1 − 𝑏 𝑘 )
𝑛→∞ 𝑛 𝑛→∞ 𝑛 𝑛→∞ 𝑛
𝑘=1 𝑘=1 𝑘=1

𝑛
!
1 Õ
⇒ ℓ = lim 𝑎 1 + (𝑛 + 1)𝑏 𝑛 − 2 · 𝑏1 − 𝑏 𝑘+1 .
𝑛→∞ 𝑛
𝑘=1

𝑎1 𝑏1
Como 𝑎1 = 𝑏1 são valores reais, temos que lim = lim = 0, ficando apenas com
𝑛→∞ 𝑛 𝑛→∞ 𝑛

𝑛
" #
(𝑛 + 1)𝑏 𝑛 1Õ
ℓ = lim − 𝑏 𝑘+1 .
𝑛→∞ 𝑛 𝑛
𝑘=1
9
𝑛

Por Stolz-Cesàro, temos que lim 𝑏 𝑘+1 = lim 𝑏 𝑛+1 = 𝑏, e segue que
𝑛→∞ 𝑛 𝑛→∞
𝑘=1

ℓ = lim 𝑏 𝑛 − 𝑏 𝑛+1 = 0.
𝑛→∞

Como querı́amos demonstrar.

12. Vamos primeiramente provar por indução que 𝑎 𝑛 > 0 para todo 𝑛 inteiro não negativo. Veja 𝑎0 > 0, e
supondo que para 𝑚 ⩾ 0 temos que 𝑎 𝑚 > 0, segue que 𝑎 𝑚+1 = 𝑎 𝑚 + √𝑘 1𝑎 > 0, como querı́amos demonstrar.
𝑚
Observe que 𝑎 𝑛+1 − 𝑎 𝑛 = √𝑘 1𝑎 > 0, então (𝑎 𝑛 )𝑛 ⩾0 é uma sequência estritamente crescente.
𝑛
Vamos assumir por absurdo que 𝑎 𝑛 converge, seja ℓ o valor real para o qual 𝑎 𝑛 converge. Então
1 1
ℓ = ℓ + √𝑘 ⇒ √𝑘 = 0.
ℓ ℓ
Absurdo, portanto, 𝑎 𝑛 → ∞. E dessa forma
𝑎 𝑛+1 1
lim = 1 + lim √ = 1.
𝑛→∞ 𝑎𝑛 𝑛→∞ 𝑎 𝑛 · 𝑘 𝑎 𝑛

Finalmente indo para o nosso limite, temos que


𝑘+1 𝑘 𝑘+1 𝑘
𝑎 𝑘+1 ©𝑎 𝑘 ª 𝑎 𝑘 ª
𝐿 = lim 𝑛 𝑘 = lim ­ 𝑛 ® = ­ lim 𝑛 ® .
©
𝑛→∞ 𝑛 𝑛→∞ 𝑛 𝑛→∞ 𝑛
« ¬ « ¬
Por Stolz-Cesàro temos que
𝑘
q 𝑘
𝑘+1
𝑎 𝑛𝑘+1 ª
𝑘
p𝑘
© 𝑎 𝑛+1 − 𝑘+1
𝑎 𝑛+1 − 𝑎𝑛
𝑘+1
© ª
𝐿 = ­­
­ ®
® = ­ lim  q
­ ®
𝑛+1−𝑛 ® 𝑛→∞
 𝑘− 1  q  𝑘− 2 q   𝑘−2   𝑘−1
®
𝑘+1 𝑘+1 𝑘 𝑘+1 𝑘+1 𝑘 𝑘+1 𝑘 𝑘+1
𝑘 𝑘
p 𝑘
p p
𝑎 𝑛+ 𝑎 𝑎 𝑎 𝑎 𝑎
­ ®
« ¬ 1 + 𝑛+1 𝑛 + · · · + 𝑛+1 𝑛 + 𝑛
« ¬
𝑘

𝑘 𝑘−1 𝑘−1 + 𝑎 𝑘
(𝑎 𝑛+1 − 𝑎 𝑛 ) 𝑎 𝑛+ 1 + 𝑎 𝑛+1 𝑎 𝑛 + · · · + 𝑎 𝑛+1 𝑎 𝑛
©  ª
­ 𝑛 ®
= ­ lim  q
­
𝑘− 𝑘−
®
𝑛→∞ 1 2 𝑘− 𝑘−
  
2 1
q q     ®
𝑘+1 𝑘+1 𝑘 𝑘+1 𝑘+1 𝑘 𝑘+1 𝑘 𝑘+1
𝑘 𝑘
p 𝑘
p p
𝑎 𝑛+ 𝑎 𝑎 𝑎 𝑎 𝑎
­ ®
1 + 𝑛+1 𝑛 + · · · + 𝑛+1 𝑛 + 𝑛
« ¬
𝑘
© ª
𝑘 𝑘−1 𝑘
𝑎 𝑛+1 + 𝑎 𝑛+1 𝑎 𝑛 + · · · + 𝑎 𝑛
­ ®
® .
­ ®

­
 𝑘−1  q  𝑘−2 p !
­ √𝑘
q p  𝑘−1 ®®
𝑘+1
­ 𝑎 𝑛 𝑘 𝑎 𝑛+ 𝑘+1 𝑘 𝑘+1 𝑘 𝑘+1
1 + 𝑘 𝑎 𝑛+ 1 𝑎𝑛 + · · · + 𝑎𝑛 ®
« ¬
Dividindo o numerador e o denominador por 𝑎 𝑛𝑘 , nós obtemos
𝑘  𝑘−1 𝑘
𝑎 𝑛+1 𝑎 𝑛+1
 
+ +···+1
𝑎𝑛 𝑎𝑛
© ª
𝐿 = ­­ lim
­ ®
®
­𝑛→∞  1  𝑘
(𝑘−1)(𝑘+1)  
(𝑘−1)(𝑘+1) (𝑘−2)(𝑘+1) 𝑘+1 (𝑘+1)(𝑘−1) ®
𝑎 𝑛+1 𝑘 + 𝑎 𝑛+1 𝑘 𝑎 𝑛 𝑘 + · · · + 𝑎𝑛 𝑘
®
𝑎𝑛
« ¬
𝑘  𝑘−1 𝑘
𝑎 𝑛+1 𝑎 𝑛+1
 
© + +···+1 ª 𝑘
𝑎𝑛 𝑎𝑛 𝑘+1
­ ® 
⇒ 𝐿 = ­ lim ® =
­ ®
­𝑛→∞   (𝑘−1)(𝑘+ 1)  (𝑘−2)(𝑘+ 1)
𝑘
𝑎 𝑛+1 𝑎 𝑛+1
𝑘
 𝑘
®
+ +···+1
­ ®
« 𝑎𝑛 𝑎𝑛 ¬
10
𝑘
𝑎 𝑛𝑘+1

1
⇔ lim = 1+ .
𝑛→∞ 𝑛𝑘 𝑘

13. Vamos fazer alguns casos para descobrir o padrão:


∫ 𝑥
𝐹1 (𝑥) = ln (𝑡) 𝑑𝑡 = 𝑥 ln (𝑥) − 𝑥 = 𝑥 (ln (𝑥) − 1) ;
0

𝑥
𝑥2 1 𝑥2 𝑥2
∫   
1
𝐹2 (𝑥) = 𝑡 (ln (𝑡) − 1) 𝑑𝑡 = (ln (𝑥) − 1) − · = ln (𝑥) − 1 + ;
0 2 2 2 2! 2

𝑥
𝑡2 𝑥3 1 𝑥3
∫      
1 1
𝐹3 (𝑥) = ln (𝑡) − 1 + 𝑑𝑡 = ln (𝑥) − 1 + − ·
0 2! 2 3! 2 3 3!
𝑥3
  
1 1
= ln (𝑥) − 1 + + .
3! 2 3

Ou seja, conjecturamos que


𝑥𝑛
𝐹𝑛 (𝑥) =
(ln (𝑥) − 𝐻𝑛 ) ,
𝑛!
em que 𝐻𝑛 é a série harmônica. Suponhamos por hipótese que tal igualdade seja verdadeira, então
𝑥
𝑡𝑛 𝑥 𝑛+1 𝑥 𝑛+1

1
𝐹𝑛+1 (𝑥) = (ln (𝑡) − 𝐻𝑛 ) 𝑑𝑡 = (ln (𝑥) − 𝐻𝑛 ) − ·
0 𝑛! (𝑛 + 1)! 𝑛 + 1 (𝑛 + 1)!
𝑥 𝑛+1
= (ln (𝑥) − 𝐻𝑛+1 ) .
(𝑛 + 1)!
𝐻𝑛
Provamos por indução portanto, que tal igualdade é valida, logo, 𝐹𝑛 (1) = − , e dessa forma:
𝑛!
𝑛!𝐹𝑛 (1) −𝐻𝑛
lim = lim .
𝑛→∞ ln (𝑛) 𝑛→∞ ln (𝑛)

Para 𝑛 suficientemente grande temos que 𝐻𝑛 = ln (𝑛) + 𝛾, em que 𝛾 é a constante de Euler-Mascheroni, e


concluı́mos que
𝑛!𝐹𝑛 (1) ln (𝑛) + 𝛾
lim = − lim = −1.
𝑛→∞ ln (𝑛) 𝑛→∞ ln (𝑛)
Uma outra forma de calcular o limite é usando Stolz-Cezàro:
𝑛 1

𝑛!𝐹𝑛 (1) 𝐻𝑛 1 Õ1 𝑛+1
lim = − lim = − lim = − lim
𝑛→∞ ln (𝑛) 𝑛→∞ ln (𝑛) 𝑛→∞ ln (𝑛) 𝑘 𝑛→∞ ln 𝑛+1

𝑛 𝑘=1
1 1
= − lim =− = −1.
𝑛→∞ 1 𝑛+1 ln (𝑒)

ln 1 + 𝑛

2.1 Referências bibliográficas


• Stolz–Cesàro theorem; From Wikipedia, the free encyclopedia;
• Tá Estudando? Cálculo - Sandro Davison;
• Análise Real - vol.1 - Funções de uma Variável - Elon Lages Lima.

11
12

Você também pode gostar